How Do You Solve a Laplace Transform with Partial Fractions?

  • Thread starter Thread starter ihatelaplace
  • Start date Start date
  • Tags Tags
    Partial
Click For Summary
The discussion revolves around solving a Laplace transform involving a partial fraction, specifically (s-1) / (s^2 + 2s + 2). The user is attempting to apply partial fraction decomposition but is uncertain about their approach and the resulting expression. Another participant suggests using the characteristic equation to find roots in the form of p ± iq, which can lead to a more straightforward solution. The conversation highlights the importance of correctly applying methods for solving differential equations using Laplace transforms. Ultimately, the focus is on clarifying the steps needed to achieve the correct solution.
ihatelaplace
Messages
1
Reaction score
0

Homework Statement



Got stuck doing the transform on a partial fraction that's giving me trouble

Homework Equations



y" + 2y' + 2y = 0 IC: F(0) = 1 F'(0) = -3

The Attempt at a Solution



Im getting stuck with a partial fraction of (s-1) / (s^2 + 2s + 2)
the only thing i can think of is As+b = 1s-1 A = 1 B = -1 and i know if i change D(s) to (s+1)^2 +1 i can get e^(-t).
(s - a)^2 + b^2 means a = -1 and b = +/- 1 then As+b / s^2 + 2s + 2 = [b - (s + a)] / [(s-a)^2 +b^2] and i get e^-t * [sin(-t) - cos(-t)] I don't think it is right tho
 
Physics news on Phys.org
Welcome to PF!

Hi ihatelaplace! Welcome to PF! :smile:

(try using the X2 tag just above the Reply box :wink:)

Your solution is nearly right, but I don't follow how you got there. :confused:

Why not just use the standard -b ± √(b2 - ac) solution to the characteristic equation …

that'll give you roots of the form p ± iq, for which the solutions are … ? :smile:
 
Question: A clock's minute hand has length 4 and its hour hand has length 3. What is the distance between the tips at the moment when it is increasing most rapidly?(Putnam Exam Question) Answer: Making assumption that both the hands moves at constant angular velocities, the answer is ## \sqrt{7} .## But don't you think this assumption is somewhat doubtful and wrong?

Similar threads

  • · Replies 6 ·
Replies
6
Views
1K
Replies
9
Views
2K
  • · Replies 7 ·
Replies
7
Views
2K
  • · Replies 1 ·
Replies
1
Views
1K
  • · Replies 1 ·
Replies
1
Views
1K
  • · Replies 6 ·
Replies
6
Views
2K
  • · Replies 18 ·
Replies
18
Views
3K
  • · Replies 2 ·
Replies
2
Views
2K
Replies
2
Views
1K
  • · Replies 8 ·
Replies
8
Views
1K